subject
Physics, 07.05.2021 05:00 Slay2020

A circuit contains four resistors connected in series. What happens to the equivalent resistance when one of the resistors is replaced with an ideal wire?

ansver
Answers: 3

Other questions on the subject: Physics

image
Physics, 22.06.2019 17:00, marchellepenuliar
Sawyer is studying diffraction. he draws a diagram of a plane wave to show how light waves travel. which best describes sawyer’s error? the wave fronts should be perpendicular to the direction in which the waves move. the arrow showing the direction of movement of the waves should be pointing to the left. the arrow showing the direction of movement of the waves should be pointing downward. the wave fronts should be both parallel and perpendicular to the direction in which the waves move.
Answers: 3
image
Physics, 23.06.2019 00:30, croxy0514
Which of the following are forms of phase changes a. freezing b. condensation c. heating d. exchanging
Answers: 2
image
Physics, 23.06.2019 02:30, brodycruce
Afaulty model rocket moves in the xy-plane (the positive y direction is vertically upward). the rocket's acceleration has components ax(t) = ? t2 and ay(t) = ? ? ? t, where ? = 2.50 m/s4 , ? = 9.00 m/s2 , and ? = 1.40 m/s3 . at t = 0 the rocket is at the origin and has velocity ~v0 = v0x ? i + v0y ? j with v0x = 1.00 m/s and v0y = 7.00 m/s. (a) calculate the velocity and position vectors as functions of time. (b) what is the maximum height reached by the rocket? (c) sketch the path of the rocket. (d) what is the horizontal displacement of the rocket when it returns to y = 0?
Answers: 1
image
Physics, 23.06.2019 03:40, hernsl0263
Anumber of conditions are required for a population to be in hardy-weinberg equilibrium. which of the following are correct descriptions of the conditions that must be met? check all that apply. a. no mutations b. random mating c. small population d. migration or gene flow e. occurrence of mutations f. no natural selection
Answers: 1
You know the right answer?
A circuit contains four resistors connected in series. What happens to the equivalent resistance whe...

Questions in other subjects:

Konu
Mathematics, 30.03.2020 18:18